K
Khách

Hãy nhập câu hỏi của bạn vào đây, nếu là tài khoản VIP, bạn sẽ được ưu tiên trả lời.

9 tháng 1 2017

ĐKXĐ : \(x\ne-1\)

Ta có \(\frac{x^4+1}{\left(x^2+1\right)\left(x+1\right)^2}=\frac{17}{45}\Leftrightarrow\frac{\left(x^2+1\right)^2-2x^2}{\left(x^2+1\right)\left(x^2+1+2x\right)}=\frac{17}{45}\)

Đặt \(a=x^2+1\), \(b=x\) thì PT đã cho trở thành

\(\frac{a^2-2b^2}{a\left(a+2b\right)}=\frac{17}{45}\) \(\Leftrightarrow2\left(2a-5b\right)\left(7a+9b\right)=0\)

Tới đây bạn tự giải đc rồi nhé :)

23 tháng 3 2020

\(\left\{{}\begin{matrix}x-\frac{3}{4}y=0\\\frac{1}{2}\left(x+3\right)\left(y-3\right)=\frac{1}{2}xy+12\end{matrix}\right.\)

\(\Leftrightarrow\left\{{}\begin{matrix}x=\frac{3}{4}y\\\frac{1}{2}\cdot\left(\frac{3}{4}y+3\right)\left(y-3\right)=\frac{1}{2}\cdot\frac{3}{4}y\cdot y+12\left(1\right)\end{matrix}\right.\)

\(\left(1\right)\Leftrightarrow\frac{3y^2}{8}+\frac{3y}{8}-\frac{9}{2}=\frac{3y^2}{8}+12\)

\(\Leftrightarrow\frac{3y}{8}=\frac{33}{2}\)

\(\Leftrightarrow y=44\)

\(\Leftrightarrow x=\frac{3}{4}\cdot44=33\)

Vậy...

29 tháng 9 2016

a/ Ta luôn có : \(\begin{cases}x^2\ge0\\\left(y-\frac{1}{10}\right)^4\ge0\end{cases}\)\(\Rightarrow x^2+\left(y-\frac{1}{10}\right)^4\ge0\)

Để dấu "=" xảy ra thì x = 0 , y = 1/10

b/ Tương tự.

27 tháng 3 2020

\(\left(x-3\right)^3-2\left(x-1\right)=x\left(x-2\right)^2-5x^2\)

\(\Leftrightarrow x^3-9x^2+27x-27-2x+2=x^3-4x^2+4x-5x^2\)

\(\Leftrightarrow27x-2x-4x-27+2=0\)

\(\Leftrightarrow21x=25\)

\(\Leftrightarrow x=\frac{25}{21}\)

Hết ý tưởng,phá tung ra,sai chỗ nào tự sửa nhé !

27 tháng 3 2020

\(\frac{\left(x+1\right)^2}{3}+\frac{\left(x+2\right)\left(x-3\right)}{2}=\frac{\left(5x-1\right)\left(x-4\right)}{6}+\frac{28}{3}\)

\(\Leftrightarrow\frac{2\left(x+1\right)^2+3\left(x+2\right)\left(x-3\right)-\left(5x-1\right)\left(x-4\right)}{6}=\frac{28}{3}\)

\(\Leftrightarrow\frac{2x^2+4x+2+3x^2-3x-18-5x^2-21x+4}{6}=\frac{28}{3}\)

\(\Leftrightarrow\frac{\left(4x-3x-21x\right)+\left(2-18+4\right)}{6}=\frac{56}{6}\)

\(\Leftrightarrow-20x-12=56\)

\(\Leftrightarrow-20x=68\)

\(\Leftrightarrow x=-\frac{17}{5}\)

Tự check lại nhá

24 tháng 9 2016

câu a tớ giải được rồi, các bn giải câu b giùm mk

13 tháng 7 2017

Để \(\frac{2x\left(3x-5\right)}{x^2+1}< 0\)

ta thấy x2+1 luôn dương với mọi x 

nên 2x(3x-5) <0

TH1: \(\orbr{\begin{cases}2x< 0\\3x-5>0\end{cases}\Leftrightarrow\orbr{\begin{cases}x< 0\\3x>5\end{cases}\Leftrightarrow}\orbr{\begin{cases}x< 0\\x>\frac{5}{3}\end{cases}\left(ktm\right)}}\)

TH2: \(\orbr{\begin{cases}2x>0\\3x-5< 0\end{cases}\Leftrightarrow\orbr{\begin{cases}x>0\\3x< 5\end{cases}\Leftrightarrow}\orbr{\begin{cases}x>0\\x< \frac{5}{3}\end{cases}\left(tm\right)}}\)

vậy \(0< x< \frac{5}{3}\)

 THẤY ĐÚNG CHO MK 1 NẾU KO HIỂU THÌ ib NHA

28 tháng 3 2018

\(\frac{2x\left(3x-5\right)}{x^2+1}< 0\)

\(\Rightarrow2x\left(3x-5\right)< 0\)  ( vì \(x^2+1>0\))

\(\Rightarrow\hept{\begin{cases}2x< 0\\3x-5>0\end{cases}}\)  hoặc \(\hept{\begin{cases}2x>0\\3x-5< 0\end{cases}}\)

\(\Rightarrow\hept{\begin{cases}x< 0\\x>\frac{5}{3}\end{cases}}\)  hoặc \(\hept{\begin{cases}x>0\\x< \frac{5}{3}\end{cases}}\)

\(\Rightarrow0< x< \frac{5}{3}\)

28 tháng 3 2020

a) ĐKXĐ: x khác +2

\(\frac{x-2}{2+x}-\frac{3}{x-2}-\frac{2\left(x-11\right)}{x^2-4}\)

<=> \(\frac{x-2}{2+x}-\frac{3}{x-2}=\frac{2\left(x-11\right)}{\left(x-2\right)\left(x+2\right)}\)

<=> (x - 2)^2 - 3(2 + x) = 2(x - 11)

<=> x^2 - 4x + 4 - 6 - 3x = 2x - 22

<=> x^2 - 7x - 2 = 2x - 22

<=> x^2 - 7x - 2 - 2x + 22 = 0

<=> x^2 - 9x + 20 = 0

<=> (x - 4)(x - 5) = 0

<=> x - 4 = 0 hoặc x - 5 = 0

<=> x = 4 hoặc x = 5

làm nốt đi 

11 tháng 6 2019

\(P=\frac{\left(x+\frac{1}{x}\right)^6-\left(x^6+\frac{1}{x^6}\right)-2}{\left(x+\frac{1}{x}\right)^3-\left(x^3+\frac{1}{x^3}\right)}\)

\(=\frac{\left(x+\frac{1}{x}\right)^6-\left[\left(x^3\right)^2+2x^3\cdot\frac{1}{x^3}+\left(\frac{1}{x^3}\right)^2\right]}{\left(x+\frac{1}{x}\right)^3-\left(x^3+\frac{1}{x^3}\right)}\)

\(=\frac{\left(x+\frac{1}{x}\right)^6-\left(x^3+\frac{1}{x^3}\right)^2}{\left(x+\frac{1}{x}\right)^3-\left(x^3+\frac{1}{x^3}\right)}\)

\(=\frac{\left[\left(x+\frac{1}{x}\right)^3-\left(x^3+\frac{1}{x^3}\right)\right]\left[\left(x+\frac{1}{x}\right)^3+\left(x^3+\frac{1}{x^3}\right)\right]}{\left(x+\frac{1}{x}\right)^3-\left(x^3+\frac{1}{x^3}\right)}\)

\(=\left(x+\frac{1}{x}\right)^3+\left(x^3+\frac{1}{x^3}\right)\ge\left(2\sqrt{x\cdot\frac{1}{x}}\right)^3+2\sqrt{x^3\cdot\frac{1}{x^3}}=8+2=10\)

Dấu "=" khi x = 1